Search found 151 matches


C

F(-1.6) = -1.96
after 3 units left and 1 unit up, it'll be -.96

by PussInBoots

Sat Feb 13, 2010 12:29 pm
Forum: Problem Solving
Topic: the consequential graph
Replies: 3
Views: 1505

Sorry for bringing this thread from archive, but what's wrong with option B?

edit: begun vs began, got it. F!@#king irregular verb sneaked in :)

by PussInBoots

Tue Oct 20, 2009 2:16 pm
Forum: Sentence Correction
Topic: OG12 / 111
Replies: 8
Views: 5189

(3^27)(5^10)(z) = (5^8)(9^14)(x^y)
25z = 3 x^y
z = 3 * x^y / 25 -> x is multiple of 5
(2) alone says that x = 5, hence z = 3 * 5^y / 25, still not enough info cuz y = 2 and y = 4 work too. (1) narrows our choices to y = 2.

Answer is C

by PussInBoots

Wed Sep 16, 2009 2:05 pm
Forum: Data Sufficiency
Topic: MGmat number properties
Replies: 7
Views: 1969

Remember formula: x*y = GCF(x,y) * LCM(x,y) (ManhattanGMAT)
xy = 1000, clearly divisible by 8.

by PussInBoots

Wed Sep 16, 2009 12:15 pm
Forum: Data Sufficiency
Topic: Multiple of 8
Replies: 2
Views: 1682

2*4*6*8*..*100 = 1*2 * 2*2 * 3*2 * 4*2 *... * 50*2 = 50! * 2^50 50! is divisible by any positive integer less or equal to 50, just like 4! is divisible by 1, 2, 3, 4. Hence 50! * 2^50 is divisible by any positive integer less or equal to 50. If you add 1, then the new number is not divisible by any ...

by PussInBoots

Wed Sep 16, 2009 12:09 pm
Forum: Problem Solving
Topic: Function/Prime Number Problem
Replies: 4
Views: 1620

woo wrote:IMO E becuase you don't know whether A is independent from B.

by PussInBoots

Mon Sep 14, 2009 11:30 pm
Forum: Data Sufficiency
Topic: probability
Replies: 3
Views: 1356

Don't forgetm, Friday is 2nd greatest. It's B.

by PussInBoots

Mon Sep 14, 2009 11:25 pm
Forum: Data Sufficiency
Topic: 90 copies of book
Replies: 4
Views: 1523
by PussInBoots

Mon Sep 14, 2009 11:21 pm
Forum: Data Sufficiency
Topic: Sqaure of an integer
Replies: 7
Views: 1842

1. 8 vs 16 2. k = p1 * p2 * p3 * ... * pK p1, p2, .., pK are prime numbers in prime factorization "exactly different prime numbers" -> p1 through pK are different (to my understanding) hence k cannot be square of integer. Square of integers must have even numver of each prime number in its...

by PussInBoots

Mon Sep 14, 2009 11:21 pm
Forum: Data Sufficiency
Topic: Sqaure of an integer
Replies: 7
Views: 1842

10^50 has fifty zeros.

Subtract 1 and we get fifty nines.
Subtract 74 and we get 48 nines, 2 and 6.

48 * 9 + 2 + 6 = 360 + 72 + 8 = 440

Answer is C

http://www.beatthegmat.com/base-10-nota ... 20304.html

by PussInBoots

Mon Sep 14, 2009 11:11 pm
Forum: Problem Solving
Topic: Integer notation
Replies: 5
Views: 1457

I disagree

Total combinations: 4*4*4*4
Total combinations of different numbers: 4*3*2*1

Answer is 4! / 4^4 = 1/8, hence ~12%

by PussInBoots

Sun Sep 13, 2009 10:26 pm
Forum: Problem Solving
Topic: As part of a game, four people each must secretly choose an
Replies: 16
Views: 4193

The answer is E. x and y has to be opposite signs, hence E is always correct.

by PussInBoots

Sat Sep 12, 2009 6:02 pm
Forum: Problem Solving
Topic: Number properties
Replies: 6
Views: 1367

E is correct, but it's not the best choice because it's too wordy.

by PussInBoots

Sat Sep 12, 2009 12:26 pm
Forum: Sentence Correction
Topic: Not only!
Replies: 4
Views: 1700
by PussInBoots

Sat Sep 12, 2009 12:10 pm
Forum: Sentence Correction
Topic: pl solve ths..exam next week
Replies: 1
Views: 1126

IMO B

C,D,E are definitely wrong, A sound awkward

by PussInBoots

Sat Sep 12, 2009 12:01 pm
Forum: Sentence Correction
Topic: swimming
Replies: 9
Views: 7286